Intervallo aperto come unione di chiusi

Analisi, algebra lineare, topologia, gruppi, anelli, campi, ...
Rispondi
Avatar utente
mates
Messaggi: 65
Iscritto il: 01 gen 1970, 01:00

Intervallo aperto come unione di chiusi

Messaggio da mates »

Carino, da un corso di Analisi 1 :

E' possibile decomporre l'intervallo aperto (0,1) come unione disgiunta di intervalli chiusi di lunghezza (misura) positiva ?

Buon lavoro
mates
killing_buddha
Messaggi: 209
Iscritto il: 20 mag 2007, 12:39

Messaggio da killing_buddha »

Il contrario so che non è vero. Due (o n) aperti non fanno un chiusolimitato. Forse si fa nello stesso modo, per assurdo? :)
Avatar utente
edriv
Messaggi: 1638
Iscritto il: 16 feb 2006, 19:47
Località: Gradisca d'Isonzo
Contatta:

Messaggio da edriv »

Se, killing_buddha, vuoi pensare seriamente a questo problema potresti leggere un hint:

Dato il nostro insieme di chiusi disgiunti, vogliamo cercare un buco tra di loro. Come? Magari costruendo due successioni di chiusi che cercano di spiaccicarsi verso un punto, sia da destra sia da sinistra.
Tibor Gallai
Messaggi: 1776
Iscritto il: 17 nov 2007, 19:12

Messaggio da Tibor Gallai »

Ripetiamo la costruzione dell'insieme di Cantor, ma usando degli intervalli chiusi anziché aperti.
francesco90
Messaggi: 8
Iscritto il: 09 dic 2007, 22:37

Re: Intervallo aperto come unione di chiusi

Messaggio da francesco90 »

mates ha scritto:
E' possibile decomporre l'intervallo aperto (0,1) come unione disgiunta di intervalli chiusi di lunghezza (misura) positiva ?

Buon lavoro
mates
ma unione anche infinita? allora forse [1/4;3/4] ; [1/12;2/12] ; [10/12;11/12] ;... e così via prendendo sempre gli intervalli "centrali tra gli spazi vuoti che rimangono ( non so se mi sono spiegato) va bene: 0,1 non appartengono a quest'unione; invece per ogni [a,b] tra gli elencati si ha che se $ a<x<b $ allora $ x $ appartiene all'unione; $ a,b $ apprtengono all'unione e sono dei punti di accumulazione.
Avatar utente
edriv
Messaggi: 1638
Iscritto il: 16 feb 2006, 19:47
Località: Gradisca d'Isonzo
Contatta:

Messaggio da edriv »

Tibor Gallai ha scritto:Ripetiamo la costruzione dell'insieme di Cantor, ma usando degli intervalli chiusi anziché aperti.
Così facendo togli, rispetto all'insieme di Cantor, una quantità numerabile di punti, no? Ma allora ne restano tanti altri...

A francesco90: mancherebbe da dimostrare che ogni punto appartiene ad uno di quegli intervalli.
Tibor Gallai
Messaggi: 1776
Iscritto il: 17 nov 2007, 19:12

Messaggio da Tibor Gallai »

Sì, per esempio un punto che non prendo con questa costruzione è 1/4.
Mi ero accorto di questo problema di cardinalità, ma per qualche motivo mi ero convinto che la costruzione andasse bene.
Tibor Gallai
Messaggi: 1776
Iscritto il: 17 nov 2007, 19:12

Messaggio da Tibor Gallai »

Ok, come penitenza dimostro l'impossibilità della costruzione (in effetti l'ha già dimostrata edriv, mi limito ad esplicitare il suo hint, che è la parte rognosa).

Supponiamo per assurdo che esista una costruzione che risolve il problema. Con un solo intervallo è impossibile, quindi ne esistono almeno due: $ [a_1,b_1] $ e $ [a_2,b_2] $, con $ b_1<a_2 $.
Formo una successione di intervalli della costruzione $ [a_n,b_n] $, in questo modo: se $ n>2 $ è dispari, $ [a_n,b_n] $ è l'intervallo della costruzione che contiene il punto $ (b_{n-2}+a_{n-1})/2 $; se $ n>2 $ è pari, $ [a_n,b_n] $ è l'intervallo della costruzione che contiene il punto $ (b_{n-1}+a_{n-2})/2 $.
In altre parole, ad ogni passo mi sposto alternativamente a destra e a sinistra, prendendo l'intervallo della costruzione che contiene il punto medio del segmento scoperto adiacente all'intervallo preso al passo precedente.
Le successioni $ \{b_{2n+1}\} $ e $ \{a_{2n}\} $ sono monotone, rispettivamente crescente e decrescente, ed ogni punto della prima è strettamente minore di ogni punto dell'altra. Poiché nella mia costruzione considero i punti medi degli intervallini scoperti, le due successioni convergono al medesimo limite $ c $ (le distanze continuano a dimezzarsi).
Nessun intervallo della mia costruzione contiene $ c $, perché $ a_{2n+1}<b_{2n+1}<c<a_{2m}<b_{2m} $, per ogni $ n,m $. Inoltre, poiché $ c $ è limite, esistono punti della successione di intervalli arbitrariamente vicini ad esso.
Se qualche intervallo contenesse $ c $, dovrebbe intersecare anche qualche intervallo della mia costruzione, assurdo.

Modificando leggermente la dimostrazione, e variando le scelte destra-sinistra nella costruzione della successione di intervalli, si può dimostrare che ogni insieme di intervalli chiusi disgiunti lascia scoperta una quantità di punti più che numerabile.
Tibor Gallai
Messaggi: 1776
Iscritto il: 17 nov 2007, 19:12

Messaggio da Tibor Gallai »

Anzi, mi viene in mente una dimostrazione più veloce che usa un teorema classico di teoria degli ordinamenti (ma che di fatto è quasi la stessa cosa che ho detto prima).

Si consideri un insieme A di intervalli chiusi e disgiunti contenuti in (0,1). Vogliamo dimostrare che esiste una quantità più che numerabile di punti di (0,1) che non appartiene all'unione degli elementi di A.
Se A è finito, allora sia [a,b] l'intervallo più a sinistra di A. Tutti i punti dell'intervallo (0,a) sono esclusi dal ricoprimento.
Se A è infinito, allora è numerabile, perché gli elementi di A sono disgiunti, ed ognuno contiene un razionale. Se un segmento resta fuori dal ricoprimento, è fatta. Altrimenti, imponiamo la relazione d'ordine banale su A: [a,b] < [c,d] se e solo se b<c. L'insieme A con questa relazione è un ordine lineare denso senza estremi, i cui assiomi sono:

$ DLO1: \forall x \neg(x<x) $
$ DLO2: \forall x,y,z (x<y\vee y<z\rightarrow x<z) $
$ DLO3: \forall x,y (x<y\vee y<x\vee x=y) $
$ DLO4: \forall x,y(x<y\rightarrow \exists z(x<z\wedge z<y)) $
$ DLO5: \forall x \exists y,z(y<x\wedge x<z) $

In particolare, gli ultimi due sono verificati perché abbiamo escluso l'esistenza di intervalli scoperti.
E' un fatto noto e dimostrato da Cantor che un ordinamento siffatto, qualora numerabile, è isomorfo a $ \mathbb{Q} $ con l'ordinamento standard.
Non ci resta che considerare un qualsiasi irrazionale di $ \mathbb{R} $, prendere le classi contigue di razionali che lo definiscono, mandarle in A attraverso l'isomorfismo, e constatare che tra le due classi di intervalli vi è un punto scoperto (più o meno come ho fatto nel post precedente). Inolte, i punti così costruiti sono tutti distinti (sono separati da un elemento di A, poiché tra due irrazionali distinti vi è sempre un razionale, e l'ordinamento di A esteso ai nuovi punti rispetta quello di $ \mathbb{R} $), e quindi più che numerabili.
Rispondi